Find the value of k for the integral

  • Thread starter Thread starter Emethyst
  • Start date Start date
  • Tags Tags
    Integral Value
Click For Summary
To find the value of k that divides the area under the curve f(x)=sqrt(x+2), the x-axis, and the line x=2 into two equal regions, the integral from -2 to k must equal the integral from k to 2. The equation to solve is ∫_{-2}^k √(x + 2) dx = ∫_k^2 √(x + 2) dx. After some calculations, it was determined that k is approximately 0.52, which can be expressed exactly as 2(cuberoot2 - 1). This solution provides a clear method for determining k in relation to the area under the curve.
Emethyst
Messages
117
Reaction score
0

Homework Statement


Find the value of k so that the region enclosed by f(x)=sqrt(x+2), the x-axis, and the line x=2 is divided by x = k into two regions of equal area.



Homework Equations


definite integral properities, fundamental theorem of calculus



The Attempt at a Solution


I have no problem finding the integral for this area, I'm just confused over the last part of the question and the use of the line x = k. How am I supposed to bring this into the answer to find the value of k that makes the area two equal regions? I had an idea to sub k in for x, but this doesn't appear to be anywhere near the right method. Any help with this question would be greatly appreciated, thanks in advance.
 
Physics news on Phys.org
You want to find k so that
\int_{-2}^k \sqrt{x + 2} dx = \int_k^2 \sqrt{x + 2}dx

By observation, it looks like k will be somewhere around 1/2.
 
Ahh thanks for that Mark, I see how to do it now, and the answer is actually a weird decimal that in exact form is 2(cuberoot2 - 1).
 
Emethyst said:
Ahh thanks for that Mark, I see how to do it now, and the answer is actually a weird decimal that in exact form is 2(cuberoot2 - 1).
Which to two decimal places is .52, so my guess was pretty close!
 
Question: A clock's minute hand has length 4 and its hour hand has length 3. What is the distance between the tips at the moment when it is increasing most rapidly?(Putnam Exam Question) Answer: Making assumption that both the hands moves at constant angular velocities, the answer is ## \sqrt{7} .## But don't you think this assumption is somewhat doubtful and wrong?

Similar threads

  • · Replies 19 ·
Replies
19
Views
2K
  • · Replies 15 ·
Replies
15
Views
2K
  • · Replies 3 ·
Replies
3
Views
956
  • · Replies 5 ·
Replies
5
Views
2K
  • · Replies 10 ·
Replies
10
Views
2K
  • · Replies 12 ·
Replies
12
Views
2K
  • · Replies 2 ·
Replies
2
Views
2K
  • · Replies 7 ·
Replies
7
Views
2K
  • · Replies 14 ·
Replies
14
Views
2K
Replies
3
Views
2K